Rätsel zum Divergenzsatz

Etwas verwirrt mich bezüglich des Divergenzsatzes. Üblicherweise schreibt man den Divergenzsatz als

(3.22) M D 4 X G μ v μ = M D Σ μ v μ
für irgendein Vektorfeld v μ , Wo D Σ μ ist das gerichtete Flächenelement. Siehe zum Beispiel Gleichung ( 3.22 ) in A Relativist's Toolkit von Eric Poisson. Ich überlegte, den Maxwell-Lagrangian in einen Grenzbegriff umzuwandeln, vorausgesetzt, ich bin auf der Schale. Dazu haben wir
M D 4 X G F μ v F μ v = 2 M D 4 X G F μ v μ A v = 2 M D 4 X G μ ( F μ v A v ) 2 M D 4 X G μ F μ v A v
wobei ich nur die Symmetrie der Christoffel-Symbole und die Antisymmetrie des Faraday-Tensors verwendet habe. Jetzt mit der Bewegungsgleichung
μ F μ v = 0
und den Divergenzsatz sollte ich schreiben können
M D 4 X G F μ v F μ v = 2 M D Σ μ F μ v A v .
Nun können wir eine magnetische Lösung betrachten, die durch gegeben ist
A = Q ( 1 cos θ ) D ϕ
F = D A = Q Sünde θ D θ D ϕ .

Angesichts dieser Lösung haben wir

F μ v F μ v = 2 Q 2 R 4
unter der Annahme einer Metrik der Form
D S 2 = F ( R ) D T 2 + 1 F ( R ) D R 2 + R 2 ( D θ 2 + Sünde 2 θ D ϕ 2 ) .
Ich habe es eigentlich mit Reissner-Nordstrom-Schwarzen Löchern mit magnetischer Ladung zu tun, aber ich denke, meine Verwirrung gilt allgemeiner. Meine Verwirrung ist, dass, wenn ich eine Grenze konstant nehme R , oder nur eine Grenze, so dass es normal aussieht
N μ = a D T + β D R ,
dann die rechte Seite von
M D 4 X G F μ v F μ v = 2 M D Σ μ F μ v A v .
scheint trivialerweise null zu ergeben, weil F hat nur Winkelkomponenten. Ich denke jedoch, dass es sehr einfach ist, eine Region der Raumzeit zu finden M sodass die linke Seite nicht verschwindet. Was habe ich bei dieser Überlegung falsch gemacht? Ich dachte, es könnte mit der Glätte des Vektorfelds zu tun haben v μ = F μ v A v in dem wir den Satz von Stokes anwenden, aber da ich den oben angegebenen Divergenzsatz ohne Annahmen zum Vektorfeld gesehen habe, bin ich mir nicht sicher, ob dies das Problem ist. Das Vektorfeld ergibt
v = Q 2 R 4 Sünde θ ( 1 cos θ ) θ ,
Das kann man sehen v hat eine Singularität bei θ = π . Dies könnte ein Koordinatenartefakt sein, aber das ist leicht zu erkennen G μ v v μ v v hat auch diesen singulären Punkt. Aus diesem Grund halte ich es für problematisch, den Divergenzsatz darauf anzuwenden v . Verstehe ich das Problem richtig?

EDIT1: Ich erwäge eine Region M der Raumzeit, die den Ursprung nicht enthält. Da ich dies im Zusammenhang mit einem Schwarzen Loch denke, möchte ich die Singularität nicht einbeziehen R = 0 in dem Patch, über den ich integriere. Als Beispiel für dieses Verfahren siehe Gl. ( 5.15 ) von https://arxiv.org/abs/1606.08307 , wo von einer elektrischen Lösung ausgegangen wird. Wenn wir jetzt von einer elektrischen Lösung ausgehen, haben wir

A = ( Q R + Q R ) D T
F = D A = Q 2 R 2 D R D T
Wo R + ist der Ereignishorizont. Somit
F μ v F μ v = 2 Q 2 R 4 ,
was die Massenintegration bedeutet M ändert sich nur um ein Vorzeichen gegenüber der magnetischen Lösung. Dies hängt mit der elektromagnetischen Dualität zusammen. In diesem Fall führt die Verwendung des Stokes-Theorems jedoch zu einem Grenzbeitrag ungleich Null, wie es meiner Meinung nach sein sollte. Jetzt ist mein Rätsel, warum wir in einem Fall Stokes verwenden können und im anderen scheint es nicht zu funktionieren. Es stellt sich heraus, dass in diesem Fall das Vektorfeld, in dem wir Stokes anwenden, ist
v = Q R 2 ( 1 R 1 R + ) R
die überall glatt zu sein scheint, wenn wir uns das ansehen G ( v , v ) , unter Verwendung eines guten Koordinatensystems.

EDIT2: Laut Introduction to Smooth Manifolds von John M. Lee muss das Vektorfeld glatt sein. Aber wenn das der Fall ist, nehmen wir das an, wenn wir das Variationsprinzip verwenden, um die Maxwell-Gleichungen herzuleiten F μ v δ A v ist glatt, um es zu einem Grenzterm zu machen, der verschwindet. Natürlich sind wir hier abgehoben, aber das kommt mir trotzdem seltsam vor F μ v A μ erweist sich für eine rein magnetische Lösung als nicht glatt. Hast du irgendwelche Erkenntnisse?

Beachten Sie, dass eine magnetische Lösung am Ursprung eine punktförmige Quelle hat und so weiter μ F μ v verschwindet nicht, sondern ist proportional zu δ ( X ) . Tatsächlich könnten Sie wahrscheinlich Ihre Argumentation für eine elektrische punktähnliche Quelle und in einer flachen Raumzeit wiederholen, um genau dasselbe Problem zu erreichen, aber alle tangentialen Dinge zu eliminieren.
Aber ich kann eine Region in Betracht ziehen M der Raumzeit, die den Ursprung nicht enthält. Da ich dies im Zusammenhang mit einem Schwarzen Loch denke, möchte ich die Singularität nicht einbeziehen R = 0 in dem Patch, über den ich integriere. Dann verschwindet es doch, oder?

Antworten (2)

Es liegt nicht nur am Ursprung, sondern am schlecht definierten Verhalten von A entlang der Linie θ = π . D ϕ macht dort keinen Sinn und der Koeffizient, der es multipliziert, verschwindet nicht. Integrieren Sie diese Halblinie und Ihr Integral wird funktionieren.

Im flachen Raum werde ich integrieren θ aus 0 Zu θ M , R A Zu R B , und über alles ϕ und ein Zeitintervall T . Die Normale zur Grenzfläche liegt in der R Richtung überall außer der Oberfläche an θ = θ M das ist natürlich normal bis das θ Richtung. Das geht genau in die Richtung F μ v A μ , und es wird der einzige Beitrag zum Oberflächenintegral sein.

Ihr Vektor v μ = F μ v A v ist in dem θ Richtung und Größe hat

| v | = Q 2 R 3 Sünde θ ( 1 cos θ )
Jetzt ist das Oberflächenintegral
T R A R B | v | 2 π R Sünde θ M D R = 2 π T Q 2 ( 1 cos θ M ) R A R B D R R 2 4 π T Q 2 R A R B D R R 2

Wo in der letzten Zeile kann ich frei nehmen θ M = π jetzt, wo die A Feld ist weg. Dies ist genau das Volumenintegral von Q 2 / R 4 . Wir sehen die Fläche 4 π das Zeitintervall T und ein Faktor von R 2 kommt aus dem Integrationsmaß.

Vielen Dank! Zuerst verstand ich nicht, was Sie anriefen | v | aber jetzt verstehe ich, dass es nur so ist G ( v , v ) . Also ja, das macht Sinn, danke.

Wenn Sie die Maschinerie der gekrümmten Raumzeit entfernen, dann sind Sie gerade bei der üblichen Subtilität angekommen, die magnetische Monopole mit sich bringt. Das heißt, wenn Sie davon ausgehen F = D A dann hast du automatisch D F = 0 , die das Gaußsche Gesetz für den Magnetismus enthält B = 0 Verbot magnetischer Monopole. Der Wechsel in die gekrümmte Raumzeit lässt diese Gleichungen etwas schicker aussehen, ändert aber nicht wirklich die Logik.

Um Monopole zuzulassen, müssen wir entweder:

  • Wechsel zum Faserbündelformalismus, Beschreibung A als Anschluss an a U ( 1 ) bündeln Sie über Ihre Raumzeit, oder
  • Konto für den "Dirac-String", eine Singularität in A das tritt zwangsläufig auf, wenn man keine faserbündel verwendet, was in diesem fall bei auftritt θ = π .

Welche Option Sie auch wählen, die technische Lösung ist die gleiche. Wenn Sie Bundles verwenden, müssen Sie Ihre abdecken R = konst Oberfläche mit zwei Patches, und Sie werden zusätzliche Begriffe aus der Überlappung aufnehmen. Wenn Sie den Dirac-String verwenden, müssen Sie den Teil der Oberfläche ausschneiden, den der Dirac-String durchläuft, und das ergibt eine Grenze, die auf der rechten Seite des Divergenzsatzes beiträgt.

Vielen Dank knzhou! Die Dirac-String-Prozedur, auf die Sie sich beziehen, ist im Grunde diejenige, die von @octonion explizit gemacht wurde, oder? Haben Sie eine Referenz, die erklärt, wie es mit Faserbündeln hergestellt werden würde? Danke noch einmal!
@ blackhole1511 Ja, es ist dasselbe, was sie getan haben. Eine nette Erklärung für die Bündel findet sich in Weinbergs Classical Solutions in Quantum Field Theory .